karl has $400 in a savings account. the interest rate is 10%, compounded annually. what type of model best fits this situation?

Answers

Answer 1
The answers D
Hope this helps

Related Questions

Decide whether Equation A or B would be used to solve the problem.
You had $1.65. After a friend returned the money you had loaned her, you
had $3.50. How much did she give you?
a. n + 1.65 = 3.50
b. n - 1.65 = 3.50

Answers

Answer:

The correct answer is B because you had less when the friend did not give you the money. Therefore before you had 3.50-1.65

Step-by-step explanation:

5) William sold half of his comic book collection then purchased 10
more. He now has 24 comic books. How many books did William
begin with?

Answers

Answer:

14

Step-by-step explanation:

If he has a total of 24

and he just purchased 10

24-10= 14

True or False: A check is a document that directs the bank to deduct money from your account and make a payment

Answers

Answer:true

Step-by-step explanation: An order to a bank to pay a stated sum from the drawer's account, written on a specially printed form.

The answer is true.

can someone please help me

Answers

Answer

7 or 5 or both

Step-by-step explanation:

so first, you know it has to be 7 or 5 because all the other numbers are already on the chart and it M was that then you would get the same output value. now you figure out the equation. thats all I can help you with because I am dumb but hope you can figure out the equation.

At a real estate agency, an agent sold a house for $397,000. The commission rate is 4.5% for the real estate agency and the commission rate for the agent is 20% of the amount the real estate agency gets. How much did the agency make on the house? How much did the agent earn in commission?

Answers

Answer:

1. $97,265

2. $17,865

Step-by-step explanation:

Step one:

given data

cost of the house=  $397,000.

we are told that the commission is 4.5% real estate agency

and also the agent commission is 20%

Step two:

1.  How much did the agency make on the house?

this amount is 20% plus  4.5% together.

=20/100*397,000 +4.5/100*397,000

=0.2*397,000. +0.045*397,000.

=79400+17865

=97,265

The agent made a total of $97,265 on the house

2. How much did the agent earn in commission?

in commission, the rate is 4.5% which amounts to $17,865

A gas station had two pumps. Pump A dispensed 241,752 gallons. Pump B dispensed 113,916 more gallons than Pump A. About how many gallons did both pumps dispense?

Answers

Answer:

Total gallons= 597,420

Step-by-step explanation:

Giving the following information:

Pump A dispensed 241,752 gallons.

Pump B dispensed 113,916 more gallons than Pump A.

First, we will calculate the number of gallons dispensed by Pump B:

Pump B= Pumb A + 113,916

Pump B= 241,752 + 113,916

Pump B= 355,668 gallons

Now, in total:

Total gallons= 241,752 + 355,668

Total gallons= 597,420

(its part b to question 9 ) If there are 15 students who don't wear glasses how many students are there in class?​
PLZ HELP WILL GIVE BRANLIST

Answers

Answer:

25

Step-by-step explanation:

so lets write our ratios.

Students that wear glasses to the whole class:

2 : 5

Students that wear glasses to those that dont:

2 : 3

And now if there are 15 kids in the class that dont wear glasses, that means that we have to multiply the second ratio by 5 to get the amount of students that don't wear glasses to 15. So now the second ratio will be

10 : 15

10 being amount of kids with glasses, and 15 kids without glasses. Now we just have to add them to get the answer of 25.

Benny went to his local zoo that featured 6 lion exhibits. If these lion exhibits make up 25% of the zoo's exhibits, then how many exhibits does the zoo have in total?

Answers

Answer:

24

Step-by-step explanation:

if 6 exhibits are 25% of the entire zoo. You would need four times that amount to get the total number of exhibits because 25% × 4 = 100%. So, if you multiply 6 × 4 the total number of exhibits would be 24.

what difficulty did you encounter in determining polynomials?​

Answers

Answer:

I didn’t really encounter any difficulty in determining a polynomial. The issue was that numbers with negative exponents or those which had square roots in them aren’t referred to as polynomials.

The only minor issue was solving problems with numbers which had different exponents in them and needed some form of simplification.

Plssss helpppp ASAPP
will mark BRAINLIEST!!!

Answers

Answer:

I would say the last one

Step-by-step explanation:

The third works the same way but 46/17 is hard to calculate without a calculator

The answer is : C

Explanation: You can solve the problem using the elimination method, and that is what the question asks for!

MARK ME AS BRAINLIEST ANSWER

Si a un numero "X" le sumo 3 y al resultado lo multiplico por 2, me da 14 ¿cual es ese numero?

Answers

4 porque si le sumas eso al 3 da a 7 y por 2 da a 14

(-1)(-11) please solve!!

Answers

Answer:

A positive 11

Step-by-step explanation:

11 is your answer

whats 10 divided by 489

Answers

Answer:

0.02044989775

Step-by-step explanation:

What kind of solution does 3x+5=-2 have?

Answers

Answer:

[tex]x = - \frac{7}{3} [/tex]

step by step:

3x+5=-2

move constant to the right-hand side and change it's sign

3x=-2-5

Calculate the difference

3x=-7

Divided both sides of the equation by 3

Suppose cot(theta) = 5/12, where pi < theta < 3pi/2. What is sin(theta)?

A. -12/13
B. -5/13
C. 5/12
D. 12/13

Answers

Answer:

- 12 /13

Step-by-step explanation:

Given that:

Cot(θ) = 5 /12 ;

Recall ; cot θ = 1 / tan θ ; tan θ = sin θ / cos θ

Hence ;

tan θ = 1 / cot θ

tan θ = 1 ÷ (5 / 12)

tan θ = 1 * 12 /5 = 12/5

Tan θ = opposite / Adjacent = 12 /5

Sin θ = opposite / hypotenus

Hypotenus = √(opposite² + adjacent²)

Hypotenus = √12² + 5²

Hypotenus = √(144 + 25)

Hypotenus = √169

Hypotenus = 13

Hence,

Sin θ = opposite / hypotenus = 12 / 13

pi < theta < 3pi/2 lies in the 3rd quadrant ; SinΘ will be negative ;

Sin θ = - 12 /13

Answer:

A

Step-by-step explanation:

Ed2021

A random sample of 400 voters in a certain city are asked if they favor an additional 4% gasoline tax to provide badly needed revenues for street repairs. If more than 220 but fewer than 260 favor the sales tax, we shall conclude that 60% of the voters are for it.a.) Find the probability of committing a type I error if 60% of the voters favor the increased tax.b.) What is the probability of committing a type II error using this test procedure if actually 48% of the voters are in favor of the additional gasoline tax?

Answers

Answer:

A) α = 0.04136

B) β = 0.00256

Step-by-step explanation:

We are given;

Sample size; n = 400

Proportion; p = 60% = 0.6

Formula for mean is;

μ = np

μ = 400 × 0.6 = 240

Standard deviation is given by;

σ = √npq

Where q = 1 - p = 1 - 0.6 = 0.4

σ = √(400 × 0.6 × 0.4)

σ = √96

σ = 9.8

A) our null hypothesis is at p = 0.6

Probability of making a type I error means we reject the null hypothesis when it is true.

This can be expressed in reference to the question as;

α = P(x < 220) + P(x > 260) all at p = 0.6

Now,

P(x < 220) = z = (x¯ - μ)/σ = (220 - 240)/9.8 = -2.04

Also;

P(x > 260) = z = (260 - 240)/9.8 = 2.04

Now, from z-distribution table probability of a z-score of -2.04 is 0.02068.

Also, probability of z-score of 2.04 is (1 - P(z < 2.04) = 1 - 0.97932 = 0.02068

Thus;

α = 0.02068 + 0.02068

α = 0.04136

B) Type II error occurs when we fail to reject the null hypothesis even though it's false.

In this case our alternative hypothesis is at p = 48% = 0.48

Thus;

μ = np

μ = 400 × 0.48 = 192

Standard deviation is given by;

σ = √npq

Where q = 1 - p = 1 - 0.48 = 0.52

σ = √(400 × 0.48 × 0.52)

σ = √99.84

σ = 9.992

Type II error would be given by;

β = [((x1¯ - μ)/σ) < z > ((x2¯ - μ)/σ)]

β = [((220 - 192)/9.992) < z > ((260 - 192)/9.992)]

β = (2.8 < z > 6.81)

Rearranging this gives us;

β = P(z < 6.81) - P(z < 2.8)

From z-distribution tables, we have;

β = 1 - 0.99744

β = 0.00256

In this exercise, we have to calculate the gasoline rates, and the answer will be:

A)[tex]\alpha= 0.04136[/tex]

B)[tex]\beta= 0.00256[/tex]

We are given that the sample size; n = 400, proportion; p = 60% = 0.6 so the formula for mean is;

[tex]\mu= np\\\mu=(400)(0.6)= 240[/tex]

Standard deviation is given by;

[tex]\sigma= \sqrt{npq}[/tex]

Where:

[tex]q = 1 \\p= 0.6\\q-p=0.4[/tex]

[tex]\sigma= \sqrt{(400)(0.6)(0.4)} \\=\sqrt{96} = 9.8[/tex]

A)Our null hypothesis is at p = 0.6. Probability of making a type I error means we reject the null hypothesis when it is true. This can be expressed in reference to the question as:

[tex]\alpha= P(x<220)+P(x>260)[/tex]

Now,

[tex]P(x < 220) = z = (x^-- \mu)/\sigma = (220 - 240)/9.8 = -2.04\\P(x > 260) = z = (260 - 240)/9.8 = 2.04[/tex]

Now, from z-distribution table probability of a z-score of -2.04 is 0.02068. Also, probability of z-score of 2.04 is:

[tex](1 - P(z < 2.04) = 1 - 0.97932 = 0.02068[/tex]

Thus;

[tex]\alpha= 0.02068+0.02068\\\alpha= 0.04136[/tex]

B) Type II error occurs when we fail to reject the null hypothesis even though it's false. In this case our alternative hypothesis is at p = 48% = 0.48. Thus:

[tex]\mu= np\\\mu= (400)(0.48)=192[/tex]

Standard deviation is given by;

[tex]\sigma= \sqrt{npq} \\=\sqrt{(400)(0.48)(0.52)} = 9.992[/tex]

Type II error would be given by:

[tex]\beta= [((x^{-1} - \mu)/\sigma) < z > ((x^{-2} - \mu)/\sigma)]\\= [((220 - 192)/9.992) < z > ((260 - 192)/9.992)]\\= \beta = (2.8 < z > 6.81)\\[/tex]

Rearranging this gives us:

[tex]\beta = P(z < 6.81) - P(z < 2.8)\\= 0.00256[/tex]

See more about statistics at : brainly.com/question/7412334

Anyone know the answers???

Answers

Answer:

A. -80

B. 60

C. 98

D. -54

Step-by-step explanation:

ez give brainliest pls

A would be -80 b would be 60 c would be 98 and D is -54

Does this look good I made it this morning

Answers

Answer:

looks delicious

Step-by-step explanation:

Answer:

Man looks so good!

Step-by-step explanation:

Send me some in the mail. lol ☺☻♥

⎧ ​ y=4x(x−2) x+y=2 ​ One of the solutions to this system is (2,0)(2,0)left parenthesis, 2, comma, 0, right parenthesis. Find the other solution. Your answer must be exact. ((left parenthesis ,,comma ))

Answers

7.5 is the answer to your very long problem

[tex]log_{9}log_{2}log_{2}256[/tex]

Answers

Answer:

I hope it will help you...

AB = 3x + 5, AC = 9x - 13

Solve for x . I’ll give brainliest to the first person who answers:)

Answers

Answer:

x = 2

Step-by-step explanation:

I assumed that you wanted me to set them equal to each other, so that is what I did. I wasn't given any other number to go by so hopefully this is correct.

Superman needs to save Lois from the clutches of Lex Luthor. After flying for 7 seconds, he is 1708 meters
from her. Then at 11 seconds he is 1484 meters from her.
What is Superman's average rate?
meters per second
How far does Superman fly every 13 seconds?
meters
How close to Lois is Superman after 24 seconds?
meters

Answers

Step-by-step explanation:

1) rate = ( 1484 - 1708) ÷ ( 11 - 7) = - 56 m/ s

2) rate × 13 = - 56 × 13 = - 728 m per 13 s

3) rate = - 56

t = 24 s

d = - 56 × 24 = 1344 m

whole distance = 2100

7 × 56 = 392

and he flay till 1708 then whole distance = 392 + 1708

so the remaining distance between him and her

2100 - 1344 = 756 m

1) What would be another way to name plane M?
(picture attached)

A. plane KJH
B. plane J
C.plane HJG
D.plane GJN

Answers

The correct answer is C plane HJG

Convert 2x + 37y = 18 to point-slope form.

Answers

Answer:

x=−37/2 y+9

Step-by-step explanation:

2x+37y=18

2x+37y+−37y=18+−37y

2x=−37y+18

2x2=−37y+18/2

x=−37/2 y+9

If f(x) = 2x - 5, then f(4) is _____.

Answers

2(4)-5
8-5= 3

Answer is 3
f(4)=2(4)-5 so f(4) is 3

the function f(x) is defined by the set of ordered pairs {(5,-2),(-6,-2),(3,-2),(4,-2)

Answers

Answer:

Assuming it is a constant function.

[tex]f(x)=0x-2[/tex]

Step-by-step explanation:

(5,-2)

(-6,-2)

(3,-2)

(4,-2)

From the points, we can see that for any value of [tex]x[/tex], the function will give -2. The graph of this function will be a horizontal line, parallel to the [tex]x[/tex]-line passing the y-intersect point -2.

So the function might be

[tex]f(x)=0x-2[/tex]

If it is a constant function.

I'm considering that there are no discontinuities.

Answer:

Domain: x>=-6

Range: y=-2

Step-by-step explanation:

I think this is the one but I’m not entirely sure

A line passes through (1, –5) and (–3, 7). a. Write an equation for the line in point-slope form. b. Rewrite the equation in slope-intercept form.

Answers

Answer:

Point-slope form: y + 5 = 13/-4 (x-1)

Slope-intercept form:

Step-by-step explanation:

Ok, so first to find the equation in point-slope form, you need to find the slope. So, the slope would be 7- (-5)/-3-1 , which is 13/-4. That is your slope, or m. Now, for point-slope form, you have the equation y - y1 = 13/-4 (x-x1). Now, you take one of your points, say (1, -5), and you put it into the equation. So your point-slope form equation would be y - (-5) = 13/-4 (x - 1). This would simplify to y + 5 = 13/-4(x - 1).

Now for slope-intercept form, you have m, the slope, which is 13/-4. Substituting that into the equation, you have y = 13/-4x + b. To find b, you substitute one of your coordinates into the equation. For example, (1, -5). Then you have -5 = (13/-4 * 1) +b. Simplify this, and you have b = -5+ 13/-4, which is -20/4 + 13/-4, which is -20/4 - 13/4 = -33/4. So your final equation is y = 13/-4x - 33/4.

Hope that helped.

Traveling along a path from A to B, what is the ratio of the change in y to the change in x?

A. 5/3
B. 4/7
C. 3/5
D. 2/3

Answers

Answer:

Option C. is correct

Step-by-step explanation:

Slope of a Line

Suppose we know the line passes through points A(x1,y1) and B(x2,y2). The slope can be calculated with the equation:

[tex]\displaystyle m=\frac{y_2-y_1}{x_2-x_1}[/tex]

Please note the slope measures the ratio of the change in y to the change in x.

The points of the graph are A=(-3,-2) B=(7,4). Calculating the slope:

[tex]\displaystyle m=\frac{4-(-2)}{7-(-3)}[/tex]

[tex]\displaystyle m=\frac{6}{10}[/tex]

Simplifying by 2:

[tex]\displaystyle m=\frac{3}{5}[/tex]

Option C. is correct

Sal needs 3 quarts of blue paint and 2 quarts of white paint to paint his room. How many quarts of blue paint will he need to paint his parent's room, if he needs 6 quarts of white paint?

Answers

Answer:

9 quarts of blue paint

Step-by-step explanation:

the ration is 3:2

2 times 3 is 6, so multiply the 3 from the blue paint to 3 as well

The domain for the equation y = 3x + 14 is listed below. Find the range for the given domain.
D:{-5,0,4}
R:

Answers

Answer:

R:{-1,14,26}

Step-by-step explanation:

The domain of a function is all the x-values of the function. The range of a function is all the y-values of the function. Therefore, we must plug in every x-value in the function to get our outputs to create our range:

Given: y=3x+14

When x=-5:

y=3(-5)+14

y=-15+14

y=-1 (this is the first point of the range)

When x=0:

y=3(0)+14

y=0+14

y=14 (this is the second point of the range)

When x=4:

y=3(4)+14

y=12+14

y=26 (this is the last point of the range)

Therefore, our range is R:{-1,14,26}

The range for the given domain {-5, 0, 4} in the equation y = 3x + 14 is

{ -1, 14, 26 }.

We have,

To find the range for the given domain of {-5, 0, 4} in the equation

y = 3x + 14, we substitute each value of the domain into the equation and determine the corresponding range values.

When x = -5:

y = 3(-5) + 14

y = -15 + 14

y = -1

When x = 0:

y = 3(0) + 14

y = 0 + 14

y = 14

When x = 4:

y = 3(4) + 14

y = 12 + 14

y = 26

Therefore,

The range for the given domain {-5, 0, 4} in the equation y = 3x + 14 is

{ -1, 14, 26 }.

Learn more about functions here:

https://brainly.com/question/28533782

#SPJ4

Other Questions
CORRECT ANSWER GETS BRAINLIEST!A dealership tracks the correlation between the age of its used cars and asking price for a car. The regression line is y = 12,338 930x, where x is the age of the car, in years.If someone was interested in buying a used car that is about 4 years old, what would be a reasonable price for the car?$ A balloon drifts 140 m toward the west in 45 s; then the wind suddenly changes and the balloon flies 90 m toward the east in the next 25 s. a. What distance did it travel during the first 45 s? b. What distance did it travel during the next 25 s? c. What total distance did it travel? d. What was its average speed during the first 45 s? e. What was its average speed during the next 25 s? f. What was its average speed for the entire trip? g. What was its displacement during the first 45 s? h. What was its displacement during the next 25 s? i. What was its total displacement? 4. What areas did Russia add to its empirebetween the 1500s and 1900? A car initially traveling at 25 m/s speeds up to 60 m/s under a uniform acceleration of 5.0 m/s2 to avoid being hit by another car. How long does it take the car to speed up and get out of the way? Show your work. 10 is 32% of what number?a. ; 32 b. ; 31.25 c. ; 3.2d. ; 312.5 explain the deference discipline wich takes place in track events? An ant can travel at a constant speed of 980 inches every 5 minutes. How far does the ant travel in 1 minute? At this rate, how far can the ant travel in 7 minutes? Your answer: Which is a long-term effect of sun exposure? A 0.300 kg ball, moving with a speed of 2.5 m/s, has a head-on collision with at 0.600 kg ball initially at rest. Assuming a perfectly elastic collision, what will be the velocity of the small ball if the heavier ball has a speed of 2m/s after collision. What is the constant of proportionality? Consider the graph below.N1-3 -2 -1123453-4Which of the following inequalities defines the shaded region of the graph above?v> 3xy 2 3x + 1y 23x - 1y 21 - 3x 2 part question!122.53 rounded to the nearest whole number.18.59 rounded to the nearest whole number. GUYS ANSWER THIS QUESTION PLEASE IM LITERALLY TIMED IM BEGGING SOMEONE PLEASE PLEASE (ASAP PLZ) Which statement BEST describes diversity in teams?1.Diversity can create conflict so if at all possible teams should minimize team diversity.2.Diversity brings different perspectives and opinions so team members should always embrace diversity and respect differences.3.Diversity can add value to teams but each individual only needs to embrace his or her own culture.4.Diversity is the one aspect of a team that is not important. When a space shuttle was launched, the astronauts on board experienced an acceleration of 29.0 m/s2. If one of the astronauts had a mass of 60.0 kg, what net force in Newtons did the astronaut experience? Kayla and me went to the park and got three ice cream then on wedenday, we came back to get two ice cream how many ice cream did we had in all. Jordan develops an understanding that he has a mind that represents the world in potentially different ways than other people's minds. He uses this ability to explain his sister's preference for a movie he does not like, and to predict how his mother will react when she finds he ate all the cookies. Which term best describes Jordan's ability adBonus question:What are three character traits ofthe men and bops Ginger talks about:1. Lentic Systems and give examples What is the charge on an atom after it gains two electrons during the formation of a bond? One positive charge Two positive charges Two negative charges One negative charge